No more than one state of nature can occur at a given time for a chance event. This indicates that the states of nature are defined such that they are
a. conservative events.
b. mutually exclusive.
c. independent outcomes.
d. collectively exhaustive.

Answers

Answer 1

Answer:

b. mutually exclusive.

Step-by-step explanation:

The given description is an illustration of mutually exclusive events.

Take for instance, when you roll a die;

It is impossible to have an outcome of 2 and 6 at the same time; these means that 2 and 6 are mutually exclusive.

In a nutshell, when two or more sates of events/states of nature can not happen at the same time; such events/states of nature are mutually exclusive.


Related Questions

You are certain to get a heart, diamond, club, or spade when selecting cards from a shuffled deck. Express the indicated degree of likelihood as a probability value between 0 and 1 inclusive.

Answers

Answer:

The probability of this event is represented by a value of 1.

Step-by-step explanation:

Probability of a certain event:

The probability of an event that is considered to be certain, that is, guaranteed to happen, is 100% = 1.

You are certain to get a heart, diamond, club, or spade when selecting cards from a shuffled deck.

This means that the probability of this event is represented by a value of 1.

Which of the following is not true regarding the flow of information from the adjusted trial balance on the end-of-period spreadsheet?

Answers

The revenue and expense account balances flow into the income statement.

The correct statement about the flow of information from the adjusted trial balance on the end-of-period spreadsheet is A. The revenue and expense account balances flow into the income statement.

What is an Adjusted Trial Balance?

This refers to the general ledger balance after some changes have been done an account balance such as accrued expenses, depreciation, etc.

Therefore, we can see that from the complete information, the statement that is false about the adjusted trial balance on the end-of-period spreadsheet is option A because the revenue and expense account balances does not flow into the income statement.

The other options from the complete text are:

a. The revenue and expense account balances flow into the income statement.b. The asset and liability account balances flow into the retained earnings statement.c. The revenue and expense account balances flow into the retained earnings statement.d. The retained earnings and dividends account balances flow into the balance sheet.


Read more about adjusted trial balance here:

https://brainly.com/question/14476257

#SPJ6

Help me please with this maths question thank you

Answers

Answer:

Step-by-step explanation:

A)

The opposite sides of a rectangle are equal. The width make this obvious because both of them are x.

B)

The lengths are not so obvious, but it is never the less true. The two sides are obvious and they are therefore true.

4x + 1 = 2x + 12                   Subtract 1 from both sides.

   - 1              -1

4x = 2x + 11                         Subtract 2x from both sides

-2x   -2x

2x  =  11                               Divide by 2

x = 11/2

x = 5.5

C)

P = L + L + w + w

P = 4(5.5) + 1  + 2(5.5) + 12 + 5.5 + 5.5

P = 22 + 1 + 11 + 12 + 11

P = 23 + 23 + 11

P = 57

Cenntura was having fun playing poker she needed the next two cards out to be heart so she could make a flesh five cards of the same suit there are 10 cards left on the deck and three our hearts what is the probability that two cards doubt to Seterra without replacement will both be hearts answer choices are in percentage for format rounded to the nearest whole number

Answers

Answer:

7% probability that the next 2 cards are hearts.

Step-by-step explanation:

Cards are chosen without replacement, which means that the hypergeometric distribution is used to solve this question.

Hypergeometric distribution:

The probability of x successes is given by the following formula:

[tex]P(X = x) = h(x,N,n,k) = \frac{C_{k,x}*C_{N-k,n-x}}{C_{N,n}}[/tex]

In which:

x is the number of successes.

N is the size of the population.

n is the size of the sample.

k is the total number of desired outcomes.

Combinations formula:

[tex]C_{n,x}[/tex] is the number of different combinations of x objects from a set of n elements, given by the following formula.

[tex]C_{n,x} = \frac{n!}{x!(n-x)!}[/tex]

In this question:

10 cards, which means that [tex]N = 10[/tex]

3 are hearts, which means that [tex]k = 3[/tex]

Probability that the next 2 cards are hearts:

This is P(X = 2) when n = 2. So

[tex]P(X = x) = h(x,N,n,k) = \frac{C_{k,x}*C_{N-k,n-x}}{C_{N,n}}[/tex]

[tex]P(X = 2) = h(2,10,2,3) = \frac{C_{3,2}*C_{7,0}}{C_{10,2}} = 0.0667[/tex]

0.0667*100% = 6.67%

Rounded to the nearest whole number, 7% probability that the next 2 cards are hearts.

The parametric equations for the paths of two projectiles are given. At what rate is the distance between the two objects changing at the given value of t? (Round your answer to two decimal places.) x1 = 10 cos(2t), y1 = 6 sin(2t) First object x2 = 4 cos(t), y2 = 4 sin(t) Second object t = π/2

Answers

Answer:

-  [tex]\frac{4}{\sqrt{29} }[/tex]

Step-by-step explanation:

The equations for the 1st object :

x₁ = 10 cos(2t),  and  y₁ = 6 sin(2t)

2nd object :

x₂ = 4 cos(t), y₂ = 4 sin(t)

Determine rate at which distance between objects will continue to change

solution Attached below

Distance( D )  = [tex]\sqrt{(10cos2(t) - 4cos(t))^2 + (6sin2(t) -4sin(t))^2}[/tex]

hence; dD/dt = - [tex]\frac{4}{\sqrt{29} }[/tex]

If $3000 is invested at 3% interest, find the value of the investment at the end of 7 years if the interest is compounded as follows. (Round your answers to the nearest cent.)
(i) annually
(ii) semiannually
(iii) monthly
(iv) weekly
(v) daily
(vi) continuously

Answers

Answer:

annualy=$3689.62

semiannually=$3695.27

monthly=$3700.06

weekly=$3700.81

daily=$3701.00

Continuously=$3701.03

Step-by-step explanation:

Given:

P=3000

r=3%

t=7 years

Formula used:

Where,

A represents Accumulated amount

P represents (or) invested amount

r represents interest rate

t represents time in years

n represents accumulated or compounded number of times per year

Solution:

(i)annually

n=1 time per year

[tex]A=3000[1+\frac{0.03}{1} ]^1^(^7^)\\ =3000(1.03)^7\\ =3689.621596\\[/tex]

On approximating the values,

A=$3689.62

(ii)semiannually

n=2 times per year

[tex]A=3000[1+\frac{0.03}{2}^{2(4)} ]\\ =3000[1+0.815]^14\\ =3695.267192[/tex]

On approximating the values,

A=$3695.27

(iii)monthly

n=12 times per year

[tex]A=3000[1+\frac{0.03}{12}^{12(7)} \\ =3000[1+0.0025]^84\\ =3700.0644[/tex]

On approximating,

A=$3700.06

(iv) weekly

n=52 times per year

[tex]A=3000[1+\frac{0.03}{52}]^3^6 \\ =3000(1.23360336)\\ =3700.81003[/tex]

On approximating,

A=$3700.81

(v) daily

n=365 time per year

[tex]A=3000[1+\frac{0.03}{365}]^{365(7)} \\ =3000[1.000082192]^{2555}\\ =3701.002234[/tex]

On approximating the values,

A=$3701.00

(vi) Continuously

[tex]A=Pe^r^t\\ =3000e^{\frac{0.03}{1}(7) }\\ =3000e^{0.21} \\ =3000(1.23367806)\\ =3701.03418\\[/tex]

On approximating the value,

A=$3701.03

Christian and Tanae both leave Disneyland at the same time. Christian travels north at 65 mph. Tanae travels south at 55 mph. How long will it take them to be 540 miles apart? Which of the following equations would you use to solve this word problem?
65t + 55(t − 1) = 540.
65t + 55t = 540.
65t + 55(t + 1) = 540.
None of these choices are correct.

Answers

Answer:

Step-by-step explanation:

B looks like it would work.

You add speeds * time when you are travelling in opposite directions.

I don't know why you would add or subtract 1 as in A and C

120 * t = 540

t = 540/120

t = 4.5 hours.

So after 4.5 hours they are 540 miles apart.

Answer:

b

Step-by-step explanation:

What is the standard form equation of the quadratic function shown in this graph?

Answers

Answer:

A is the equation in standard form

Step-by-step explanation:

10=−4x+3x^2 solve

please help!

Answers

Answer:

-1.28 AND 2.61

Step-by-step explanation:

[tex]10= -4x+3x^2\\ 3x^2 -4x - 10 = 0\\\\[/tex]

use quadratic formula

x =  [tex]\frac{-b+\sqrt{b^{2} -4ac} }{2a}[/tex]   x =  [tex]\frac{-b-\sqrt{b^{2} -4ac} }{2a}[/tex]

Solution/X-Intercepts: -1.28 AND 2.61    

An urn contains 2 small pink balls, 7 small purple balls, and 6 small white balls.
Three balls are selected, one after the other, without replacement.
Find the probability that all three balls are purple
Express your answer as a decimal, rounded to the nearest hundredth.

Answers

Answer:

The probability is P = 0.08

Step-by-step explanation:

We have:

2 pink balls

7 purple balls

6 white balls

So the total number of balls is just:

2 + 7 + 6 = 15

We want to find the probability of randomly picking 3 purple balls (without replacement).

For the first pick:

Here all the balls have the same probability of being drawn from the urn, so the probability of getting a purple one is equal to the quotient between the number of purple balls (7) and the total number of balls (15)

p₁ = 7/15

Second:

Same as before, notice that because the balls are not replaced, now there are 6 purple balls in the urn, and a total of 14 balls, so in this case the probability is:

p₂ = 6/14

third:

Same as before, this time there are 5 purple balls in the urn and 13 balls in total, so here the probability is:

p₃ = 5/13

The joint probability (the probability of these 3 events happening) is equal to the product between the individual probabilities, so we have:

P = p₁*p₂*p₃ = (7/15)*(6/14)*(5/13)  = 0.08

19. The sum of a number m and a number n, multiplied by ninety-one 20. Forty-one times the difference when six is subtracted from a num- bera 21. A number r divided by the difference between eighty-three and ten 22. The total of a number p and twelve, divided by eighteen 23. The product of a number c and three more than the sum of nine and twelve 24. The sum of a number y and ten, divided by the difference when a number x is decreased by five. I need to convert all of them into expressions. PLEASE HELP.​

Answers

Answer:

Step-by-step explanation:

19.

The numbers are m and n

Sum of m and n = m + n

Sum is multiplied by 91 = 91 x ( m + n )

20.

Let the number be = m

Six subtracted from the number = m - 6

41 times the difference = 41 x ( m - 6)

21.

Let the number be = r

Difference between 83 and 10 = 83 - 10 = 73

[tex]The \ number\ divided \ by\ the \ difference \ = \frac{r}{73}[/tex]

22.

Total of p and 23 = p + 12

[tex]Total \ divided \ by \ 18 = \frac{p + 12 }{18}[/tex]

23.

The product of c and 3  = 3c

Sum of 9 and 12 = 21

Product is more than Sum = 3c + 21

24.

Sum of y  and 10 = y + 10

Number x decreased by 5 = x - 5

[tex]Sum \ divided \ by \ difference = \frac{ y + 10 }{x - 5}[/tex]

If you have 3/8 of one pie, what does the denominator tells you ?

Answers

Step-by-step explanation:

There was originally 8 pieces of pie.

Answer:

if you have 3/8 of one pie, the denominator tells you that the pie was divided into 8 piece.

Step 1: For each circle (A-G) in the table below, use the given information to determine the missing
information. Include supporting work showing and explaining how you found the missing information.
Circle
Center
Radius
Equation
A
(x - 9)2 + (y - 12)2 = 64
B
(-1,-17)
5
С
(-9,13)
9n
D
x2 + (0 - 1)2 = 36
E
x2 + y2 – 26x = -160
F
*2 + y2 + 22x +12y = -93
G
x2 + y2 – 10x+12y = -52

Answers

Answer:

I don't really understand the question

Step-by-step explanation:

c

Suppose that a survey was taken and it showed that 18% of online shoppers in the United States would prefer to do business only with large well-known retailers. If 2700 online shoppers were surveyed, how many are willing to do business with any size retailers?

Answers

Step-by-step explanation:

You can conclude that 82% of all shoppers will do business with any retailer of any size aslong as they are on the internet.

82% of 2700 = 0.82 * 2700 =2214

which makes the other responder correct.


ABCD is a square of side 12 cm. It is formed from two rectangles AEGD and
EBCG. H is a point on AD and F is a point on BC.
Find the area of EFGH.

Answers

Answer:72 [tex]cm^{2}[/tex]

Solution 1:

Step 1: Find EF use Pythagorean theorem

[tex]EF^{2} = EB^{2} + BF^{2}[/tex]

[tex]EF^{2} = 6^{2} + 6^{2}[/tex]

EF = [tex]\sqrt{6^{2} + 6^{2} }[/tex] = 6[tex]\sqrt{2}[/tex] cm

Step 2: The area of EFGH = [tex]EF^{2}[/tex]= [tex](6\sqrt{2} )^{2}[/tex] = 72

Solution 2: See that the area of EFGH is equal [tex]\frac{1}{2}[/tex] the area of ABCD

The area of ABCD = 12x12 = 144

Thus, the area of EFGH = 144: 2 = 72:)

Have a nice day!

1. Find the Perimeter AND Area of the figure
below.
2 ft
5 ft
2 ft
5 ft

Answers

Answer:

A = 16 ft^2

P = 20 ft

Step-by-step explanation:

P = perimeter

A = area

STEP 1: divide the shape into rectangles

Rectangle 1: 2ft*3ft

Rectangle 2: 2ft*5ft

STEP 2: Find the area of each rectangle

Equation for area of a rectangle = bh

Rectangle 1: b = 2, h = 3

Rectangle 2: b = 2, h = 5

(2 * 3) + (2 * 5)

6 + 10

16 ft^2

Now, we have to find the perimeter

STEP 1:  Find the unknown side lengths.

To find the lengths of the sides not labeled, you have to use the lengths of the sides we already know.

The length of one parallel side is 5, and the length of another parallel side is 2. The length of the unknown side starts at the same place as the top of the side length that is 5, and ends at the top of the side length that is 2. This means that we have to subtract 2 from 5 in order to find the unknown side length.

STEP 2: Add up all the side lengths

P = 2 + 5 + 5 + 2 + 3 + 3

P = 20 ft

Don't forget to label your answers!!

I hope this made sense, it's is a little hard to explain in simple terms without being able to draw, but I hope it helped.

were should i go shopping for fidgets

Answers

Answer:

Amazon

Step-by-step explanation:

You can shop from Etsy/Amazon

Zero is not a real number True or
False​

Answers

Yes because it is one of the numbers on the number line. ( the only numbers that are not considered real numbers are decimals , fractions , ratios & etc ) .

Which represents can be used to determine the slope of the linear function graphed below

Answers

I don’t see no photo

A bacteria culture initially contains 100 cells and grows at a rate proportional to its size. After an hour the population has increased to 310.
(a) Find an expression for the number of bacteria after
hours.
(b) Find the number of bacteria after 3 hours.
(c) Find the rate of growth after 3 hours.
(d) When will the population reach 10,000?

Answers

Answer:

a) The expression for the number of bacteria is [tex]P(t) = 100\cdot e^{1.131\cdot t}[/tex].

b) There are 2975 bacteria after 3 hours.

c) The rate of growth after 3 hours is about 3365.3 bacteria per hour.

d) A population of 10,000 will be reached after 4.072 hours.

Step-by-step explanation:

a) The population growth of the bacteria culture is described by this ordinary differential equation:

[tex]\frac{dP}{dt} = k\cdot P[/tex] (1)

Where:

[tex]k[/tex] - Rate of proportionality, in [tex]\frac{1}{h}[/tex].

[tex]P[/tex] - Population of the bacteria culture, no unit.

[tex]t[/tex] - Time, in hours.

The solution of this differential equation is:

[tex]P(t) = P_{o}\cdot e^{k\cdot t}[/tex] (2)

Where:

[tex]P_{o}[/tex] - Initial population, no unit.

[tex]P(t)[/tex] - Current population, no unit.

If we know that [tex]P_{o} = 100[/tex], [tex]t = 1\,h[/tex] and [tex]P(t) = 310[/tex], then the rate of proportionality is:

[tex]P(t) = P_{o}\cdot e^{k\cdot t}[/tex]

[tex]\frac{P(t)}{P_{o}} = e^{k\cdot t}[/tex]

[tex]k\cdot t = \ln \frac{P(t)}{P_{o}}[/tex]

[tex]k = \frac{1}{t}\cdot \ln \frac{P(t)}{P_{o}}[/tex]

[tex]k = \frac{1}{1}\cdot \ln \frac{310}{100}[/tex]

[tex]k\approx 1.131\,\frac{1}{h}[/tex]

Hence, the expression for the number of bacteria is [tex]P(t) = 100\cdot e^{1.131\cdot t}[/tex].

b) If we know that [tex]t = 3\,h[/tex], then the number of bacteria is:

[tex]P(t) = 100\cdot e^{1.131\cdot t}[/tex]

[tex]P(3) = 100\cdot e^{1.131\cdot (3)}[/tex]

[tex]P(3) \approx 2975.508[/tex]

There are 2975 bacteria after 3 hours.

c) The rate of growth of the population is represented by (1):

[tex]\frac{dP}{dt} = k\cdot P[/tex]

If we know that [tex]k\approx 1.131\,\frac{1}{h}[/tex] and [tex]P \approx 2975.508[/tex], then the rate of growth after 3 hours:

[tex]\frac{dP}{dt} = \left(1.131\,\frac{1}{h} \right)\cdot (2975.508)[/tex]

[tex]\frac{dP}{dt} = 3365.3\,\frac{1}{h}[/tex]

The rate of growth after 3 hours is about 3365.3 bacteria per hour.

d) If we know that [tex]P(t) = 10000[/tex], then the time associated with the size of the bacteria culture is:

[tex]P(t) = 100\cdot e^{1.131\cdot t}[/tex]

[tex]10000 = 100\cdot e^{1.131\cdot t}[/tex]

[tex]100 = e^{1.131\cdot t}[/tex]

[tex]\ln 100 = 1.131\cdot t[/tex]

[tex]t = \frac{\ln 100}{1.131}[/tex]

[tex]t \approx 4.072\,h[/tex]

A population of 10,000 will be reached after 4.072 hours.

15. The area of a triangle is 72 in the base is 12 in. Find the height.​

Answers

Answer:

[tex]hright =12[/tex]

Step-by-step explanation:

----------------------------------------

The formula to find the area of a triangle is  [tex]A=\frac{1}{2}bh[/tex]  where [tex]b[/tex] stands for the base and [tex]h[/tex] stands for the height.

But we already know the area and the base. So to find the height, let's substitute 72 for [tex]A[/tex] and 12 for [tex]b[/tex], and solve.

[tex]72=\frac{1}{2}(12)(h)[/tex]

[tex]72=6h[/tex]

Here, divide both sides by 6

[tex]12=h[/tex]

--------------------

Hope this is helpful.

Answer:

height = 12

Step-by-step explanation:

.............

what is the value of -2x²y³ when ×=2 and y=4?​

Answers

Answer:

1024

Step-by-step explanation:

Given :-

x = 2 y = 4

Value of -2x²y³

2x³ y³2 * (2)³ * (4)³2 * 8 * 64 1024

Answer:

254

Step-by-step explanation:

^ <- this is the square sign

-2x^y^3

x=2

y=4

put x values in to x place and y value in to y place.

-2(2)^2(4)^3

Find the squares and - it with 2

-2(4)(64)

2-256=254

:. the value of -2x^2y^3 =254

That the answer.

Hope this is what you asked.

If f(x) =4x2 - 8x - 20 and g(x) = 2x + a, find the value of a so that the y-intercept of the graph of the composite function (fog)(x) is (0, 25).

Answers

Answer:

The possible values are a = -2.5 or a = 4.5.

Step-by-step explanation:

Composite function:

The composite function of f(x) and g(x) is given by:

[tex](f \circ g)(x) = f(g(x))[/tex]

In this case:

[tex]f(x) = 4x^2 - 8x - 20[/tex]

[tex]g(x) = 2x + a[/tex]

So

[tex](f \circ g)(x) = f(g(x)) = f(2x + a) = 4(2x + a)^2 - 8(2x + a) - 20 = 4(4x^2 + 4ax + a^2) - 16x - 8a - 20 = 16x^2 + 16ax + 4a^2 - 16x - 8a - 20 = 16x^2 +(16a-16)x + 4a^2 - 8a - 20[/tex]

Value of a so that the y-intercept of the graph of the composite function (fog)(x) is (0, 25).

This means that when [tex]x = 0, f(g(x)) = 25[/tex]. So

[tex]4a^2 - 8a - 20 = 25[/tex]

[tex]4a^2 - 8a - 45 = 0[/tex]

Solving a quadratic equation, by Bhaskara:

[tex]\Delta = (-8)^2 - 4(4)(-45) = 784[/tex]

[tex]x_{1} = \frac{-(-8) + \sqrt{784}}{2*(4)} = \frac{36}{8} = 4.5[/tex]

[tex]x_{2} = \frac{-(-8) - \sqrt{784}}{2*(4)} = -\frac{20}{8} = -2.5[/tex]

The possible values are a = -2.5 or a = 4.5.

-8(9r - 1) - 9(-8r+2)
Simplest form

Answers

Answer:

-10

Step-by-step explanation:

Step-by-step explanation:

-8(9r-1)-9(-8r+2)-72r+8-72r-18-72r-72r+8-18-144r-10-(144r+10)

hope it helps

stay safe healthy and happy...

HELPPP PLEASEEE! I tried everything from adding to dividing, subtracting, multiplying but still no correct answer. Can someone help me out here please? I am not sure where to start either now. Thank you for your time.

Answers

You have some data points labeled by [tex]x[/tex]. They form the set {3, 5, 7}.

The mean, [tex]\bar x[/tex], is the average of these values:

[tex]\bar x = \dfrac{3+5+7}3 = \dfrac{15}3 = 5[/tex]

Then in the column labeled [tex]x-\bar x[/tex], what you're doing is computing the difference between each data point [tex]x[/tex] and the mean [tex]\bar x[/tex]:

[tex]x=3 \implies x-\bar x = 3 - 5 = -2[/tex]

[tex]x=5 \implies x-\bar x = 5-5 = 0[/tex]

[tex]x=7 \implies x-\bar x = 7 - 5 = 2[/tex]

These are sometimes called "residuals".

In the next column, you square these values:

[tex]x=3 \implies (x-\bar x)^2 = (-2)^2 = 4[/tex]

[tex]x=5 \implies (x-\bar x)^2 = 0^2 = 0[/tex]

[tex]x=7 \implies (x-\bar x)^2 = 2^2 = 4[/tex]

and the variance of the data is the sum of these so-called "squared residuals".


Last week at the business where you work, you sold 120 items.  The business paid $1 per item and sold them for $3 each.  What profit did the business make from selling the 120 items?​

Answers

Answer:

240

Step-by-step explanation:

minus how much u sold them and how much it cost to make

3-1=2

times 2 and 120

2(120)

240

If the cutoff Z score on the comparison distribution is 2.33 and the sample value has a score of 2.35 on the comparison distribution, the correct decision is to:____.
A) fail to reject the null hypothesis.
B) reject the null hypothesis.
C) accept the researc hypothesis.
D) reject the research hypothesis.

Answers

Answer:

B) reject the null hypothesis.

Step-by-step explanation:

Which complex number does not lie on the line segment plotted on the graph?

Answers

Answer:

Notice that for 3 out of the 4 numbers, there is a relationship between the x and the y coordinate of the number; for 3+i, -2i, -2-4i we have that the real part is larger by 2 from the imaginary part. Thus, the points are on the same line in the imaginary plane; they satisfy x=y+2 or Re{z}=Im{z}+2. However, 2-4i does not satisfy this equation since 2 is not equal to -4+2. Hence, this point does not belong to the line that the other 3 points define.

Step-by-step explanation:

What is the GCF of 1683t, 4085, and 68t??
O 4
O 483t
O 8
O 8837

Answers

Answer:I’m pretty sure ( not 100% thou ) the awnser would be A) 4

Which answers describe the shape below? Check all that apply.
A. Square
B. Quadrilateral
C. Rhombus
D. Trapezoid
E. Rectangle
F. Parallelogram

Answers

Answer:

b and f

Step-by-step explanation:

Other Questions
Which statement is true about quadrilateral ABCD with vertices A(2, 8), B(3, 11), C(4, 8), and D(3, 5)? Our story begins with Greg who was torn. He knew that if he did this, his company would end up producing a lot of pollution. The lake they would end up dumping their waste into would be poisoned. Nothing would ever live there again. However, if he chose the cleaner option, it would be more expensive. He would have to lay off hundreds to afford the cleaner option. People like Ben Carlson, who was home with his three daughters, oblivious that his job was currently on the line. Greg knew that either option had its faults, but Greg valued his workers more than the environment. He signed the order and condemned the lake, but saved his workers. Based on this writing, which of the following statements would the narrator probably agree with? A. While pollution is a problem, sometimes it is unavoidable. B. Pollution is not a problem that people need to worry about. C. Pollution is the greatest issue facing the world. D. People who value human life over the well-being of the planet are making a huge mistake. Select the expression that represents the following statement: add 24 to the quotient of 16 and 8. jordan wants to save to buy a car and decides to open a banking account that is offering an interest rate of 4.5% compounded annually how much will jordan have in the account after 5 years it he deposits $7,000 today? There are two possible alignments of a dipole in an external electric field where the dipole is in equilibrium: when the dipole moment is parallel to the electric field and when the dipole moment is oriented opposite the electric field.Part AAre both alignments stable? (Consider what would happen in each case if you gave the dipole a slight twist.)a) Yesb) NoPart BBased on your answer to the previous part and your experience in mechanics, in which orientation does the dipole have less potential energy?a) The arrangement with the dipole moment parallel to the electric field has less potential energy.b) The arrangement with the dipole moment opposite the electric field has less potential energy.c) Both arrangements have the same potential energy. solve the equation 11n - 17 = 49 One angle of a triangle is twice as large as another. The measure of the third angle is 60 more than that of the smallest angle. Find the measure of each angle.The measure of the smallest angle is Please help :) You have contracted to buy a house for $300,000, paying $50,000 as a down payment and taking a fully amortizing mortgage for the balance at a 5.5% annual interest rate for 30 years. What will your monthly payment (covering principal and interest) be if you make monthly installments over the next 30 years (round to the nearest dollar) find the h.c.f. if 84 and 72 What is the correct sequence of events for the life cycle of the pork tapeworm, T. solium? Arrange the following statements in chronological order On the unit circle, which of the following angles has the terminal pointcoordinates. A. 45B. 135C. 225D. 315 NEED HELP ASAP. Please show all work. A point on a rotating wheel (thin hoop) having a constant angular velocity of 200 rev/min, the wheel has a radius of 1.2 m and a mass of 30 kg. ( I = mr2 ).(a) (5 points) Determine the linear acceleration.(b) (4 points) At this given angular velocity, what is the rotational kinetic energy? What is net cash flow Select all that apply.All cells have similar structures, including _____.a plasma membranevacuolesprotoplasmchloroplastscell wallcytoplasm Life, Inc. experienced the following events in Year 1, its first year of operation: Performed counseling services for $22,000 cash. On February 1, Year 1, paid $15,000 cash to rent office space for the coming year. Adjusted the accounts to reflect the amount of rent used during the year.RequiredBased on this information alone:a. Record the events under an accounting equation.TABLE PROVIDED BELOWa.Life, Inc.Effect of Events on the Accounting Equation Assets = Stockholders EquityEventCashPrepaid Rent =Retained Earnings1. Performed Services 36,000 36,0002. Prepaid Rent (18,000) 18,000 NA3. Used Rent (18,000) (18,000) Totals 18,000 0 = 18,000b. Prepare an income statement, balance sheet, and statement of cash flows for the 2016 accounting period.Life, Inc.Income StatementFor the Year Ended December 31, 2016 Revenue 36,000 Expense 18,000 Net Income 18,000 A wire 2.80 m in length carries a current of 5.60 A in a region where a uniform magnetic field has a magnitude of 0.300 T. Calculate the magnitude of the magnetic force on the wire assuming the following angles between the magnetic field and the current. an effect english settlement in the caolinas was the migration of the:A. IroquoisB. TuscaroraC. FrenchD. Spanish State Newtons second law of motion. Derive this law mathematically. A 1000 kg vehicle moving with a speed of 20m/s is brought to rest in a distance of 50 metres. Find the acceleration and calculate the force acting on the vehicle. How are all courts expect the us Supreme Court established True or false? One theory about phototropism is that auxin triggers a reaction on the shaded side of the stem that causes cell elongation on that side of the plant.Group of answer choicesTrueFalse